next up previous


Postscript version of these notes

STAT 801 Lecture 21

Reading for Today's Lecture:

Goals of Today's Lecture:

Today's notes

The Neyman Pearson Lemma

Theorem: In testing f0 against f1 the probability $\beta$ of a type II error is minimized, subject to $\alpha \le \alpha_0$by the test function:

\begin{displaymath}\phi(x) =\left\{\begin{array}{ll}
1 & \frac{f_1(x)}{f_0(x)} >...
...bda
\\
0 & \frac{f_1(x)}{f_0(x)} < \lambda
\end{array}\right.
\end{displaymath}

where $\lambda$ is chosen so that for each $y>\lambda$

\begin{displaymath}P_0( \frac{f_1(X)}{f_0(X)} \ge y) < \alpha_0
\end{displaymath}

and

\begin{displaymath}P_0( \frac{f_1(x)}{f_0(x)}\ge \lambda) \ge \alpha_0
\end{displaymath}

and where $\gamma$ is any number chosen so that

\begin{displaymath}E_0(\phi(X)) = P_0( \frac{f_1(X)}{f_0(X)} > \lambda)
+ \gamma P_0( \frac{f_1(X)}{f_0(X)} =\lambda)
= \alpha_0
\end{displaymath}

The value of $\gamma$ is unique if $P_0( \frac{f_1(X)}{f_0(X)} = \lambda) > 0$.

Definition: In the general problem of testing $\Theta_0$ against $\Theta_1$ the level of a test function $\phi$is

\begin{displaymath}\alpha = \sup_{\theta\in\Theta_0}E_\theta(\phi(X))
\end{displaymath}

The power function is

\begin{displaymath}\pi(\theta) = E_\theta(\phi(X))
\end{displaymath}

A test $\phi^*$ is a Uniformly Most Powerful level $\alpha_0$test if

1.
$\phi^*$ has level $\alpha \le \alpha_o$

2.
If $\phi$ has level $\alpha \le \alpha_0$ then for every $\theta\in \Theta_1$ we have

\begin{displaymath}E_\theta(\phi(X)) \le E_\theta(\phi^*(X))
\end{displaymath}

Example: In the Binomial(n,p) to test p=p0 versus p1 for a p1>p0 the NP test is of the form

\begin{displaymath}\phi(x) =1(X>k)+\gamma 1(X=k)
\end{displaymath}

where we choose k so that

\begin{displaymath}P_{p_0}(X>k) \le \alpha_0 < P_{p_0}(X \ge k)
\end{displaymath}

and $\gamma\in[0,1)$ so that

\begin{displaymath}\alpha_0 = P_{p_0}(X>k) + \gamma P_{p_0}(X =k)
\end{displaymath}

This rejection region depends only on p0 and not on p1 so that this test is UMP for p=p0 against p>p0. Since this test has level $\alpha_0$ even for the larger null hypothesis it is also UMP for $p\le p_0$ against p>p0.

Example: In the $N(\mu,1)$ example to test $\mu \le \mu_0$against $\mu= \mu_1$ for some $\mu_1>\mu_0$ the NP test is of the form

\begin{displaymath}\phi(x) =1(n^{1/2}( \bar{x}-\mu_0) > z_\alpha)\, .
\end{displaymath}

Since $\bar{X}$ has a continuous distribution there is no need to think about $\gamma$. Since the rejection region does not depend on $\mu_1$ this test is UMP for $\mu=\mu_0$ against $\mu>\mu_0$. Since it has level $\alpha$ (you can check that the power function is a strictly increasing function of $\mu$) even for the larger null $\mu \le \mu_0$ it is also UMP level $\alpha$ for $\mu \le \mu_0$ against $\mu>\mu_0$.

Proof of the Neyman Pearson lemma: Given a test $\phi$ with level strictly less than $\alpha_0$ we can define the test

\begin{displaymath}\phi^*(x) = \frac{1-\alpha_0}{1-\alpha} \phi(x) + \frac{\alpha_0-\alpha}{1-\alpha}
\end{displaymath}

has level $\alpha_0$ and $\beta$ smaller than that of $\phi$. Hence we may assume without loss that $\alpha=\alpha_0$ and minimize $\beta$ subject to $\alpha=\alpha_0$. However, the argument which follows doesn't actually need this.

For each $\lambda> 0$ we have seen that $\phi_\lambda$ minimizes $\lambda\alpha+\beta$ where $\phi_\lambda=1(f_1(x)/f_0(x) \ge \lambda) $.

As $\lambda$ increases the level of $\phi_\lambda$ decreases from 1 when $\lambda=0$ to 0 when $\lambda = \infty$. There is thus a value $\lambda_0$ where for $\lambda < \lambda_0$ the level is less than $\alpha_0$ while for $\lambda > \lambda_0$ the level is at least $\alpha_0$. Temporarily let $\delta=P_0(f_1(X)/f_0(X) = \lambda_0)$. If $\delta = 0$ define $\phi=\phi_\lambda$. If $\delta > 0$ define

\begin{displaymath}\phi(x) =\left\{\begin{array}{ll}
1 & \frac{f_1(x)}{f_0(x)} >...
...0
\\
0 & \frac{f_1(x)}{f_0(x)} < \lambda_0
\end{array}\right.
\end{displaymath}

where $P_0(f_1(X)/f_0(X) < \lambda_0)+\gamma\delta = \alpha_0$. You can check that $\gamma\in[0,1]$.

Now $\phi$ has level $\alpha_0$ and according to the theorem above minimizes $\alpha+\lambda_0\beta$. Suppose $\phi^*$ is some other test with level $\alpha^* \le \alpha_0$. Then

\begin{displaymath}\lambda_0\alpha_\phi+ \beta_\phi \le \lambda_0\alpha_{\phi^*} +
\beta_{\phi^*}
\end{displaymath}

We can rearrange this as

\begin{displaymath}\beta_{\phi^*} \ge \beta_\phi +(\alpha_\phi-\alpha_{\phi^*})\lambda_0
\end{displaymath}

Since

\begin{displaymath}\alpha_{\phi^*} \le \alpha_0 = \alpha_\phi
\end{displaymath}

the second term is non-negative and

\begin{displaymath}\beta_{\phi^*} \ge \beta_\phi
\end{displaymath}

which proves the Neyman Pearson Lemma.

The examples show a phenomenon which is somewhat general. What happened was this. For any $\mu>\mu_0$ the likelihood ratio $f_\mu/f_0$ is an increasing function of $\sum X_i$. The rejection region of the NP test is thus always a region of the form $\sum X_i > k$. The value of the constant k is determined by the requirement that the test have level $\alpha_0$and this depends on $\mu_0$ not on $\mu_1$.

Definition: The family $f_\theta;\theta\in \Theta\subset R$has monotone likelikelood ratio with respect to a statistic T(X)if for each $\theta_1>\theta_0$ the likelihood ratio $f_{\theta_1}(X)
/ f_{\theta_0}(X)$ is a monotone increasing function of T(X).

Theorem: For a monotone likelihood ratio family the Uniformly Most Powerful level $\alpha$ test of $\theta \le \theta_0$(or of $\theta=\theta_0$) against the alternative $\theta>\theta_0$is

\begin{displaymath}\phi(x) =\left\{\begin{array}{ll}
1 & T(x) > t_\alpha
\\
\gamma & T(X)=t_\alpha
\\
0 & T(x) < t_\alpha
\end{array}\right.
\end{displaymath}

where $P_0(T(X) > t_\alpha)+\gamma P_0(T(X) = t_\alpha) = \alpha_0$.

A typical family where this will work is a one parameter exponential family. In almost any other problem the method doesn't work and there is no uniformly most powerful test. For instance to test $\mu=\mu_0$against the two sided alternative $\mu\neq\mu_0$ there is no UMP level $\alpha$ test. If there were its power at $\mu>\mu_0$ would have to be as high as that of the one sided level $\alpha$ test and so its rejection region would have to be the same as that test, rejecting for large positive values of $\bar{X} -\mu_0$. But it also has to have power as good as the one sided test for the alternative $\mu < \mu_0$ and so would have to reject for large negative values of $\bar{X} -\mu_0$. This would make its level too large.

The favourite test is the usual 2 sided test which rejects for large values of $\vert\bar{X} -\mu_0\vert$ with the critical value chosen appropriately. This test maximizes the power subject to two constraints: first, that the level be $\alpha$ and second that the test have power which is minimized at $\mu=\mu_0$. This second condition is really that the power on the alternative be larger than it is on the null.

Definition: A test $\phi$ of $\Theta_0$ against $\Theta_1$ is unbiased level $\alpha$ if it has level $\alpha$ and, for every $\theta\in \Theta_1$ we have

\begin{displaymath}\pi(\theta) \ge \alpha \, .
\end{displaymath}

When testing a point null hypothesis like $\mu=\mu_0$ this requires that the power function be minimized at $\mu_0$ which will mean that if $\pi$ is differentiable then

\begin{displaymath}\pi^\prime(\mu_0) =0
\end{displaymath}

We now apply that condition to the $N(\mu,1)$ problem. If $\phi$is any test function then

\begin{displaymath}\pi^\prime(\mu) = \frac{\partial}{\partial\mu} \int \phi(x) f(x,\mu) dx
\end{displaymath}

We can differentiate this under the integral and use

\begin{displaymath}\frac{\partial f(x,\mu)}{\partial\mu} = \sum(x_i-\mu) f(x,\mu)
\end{displaymath}

to get the condition

\begin{displaymath}\int \phi(x) \bar{x} f(x,\mu_0) dx = \mu_0 \alpha_0
\end{displaymath}

Consider the problem of minimizing $\beta(\mu)$ subject to the two constraints $E_{\mu_0}(\phi(X)) = \alpha_0$ and $E_{\mu_0}(\bar{X} \phi(X)) = \mu_0 \alpha_0$. Now fix two values $\lambda_1>0$ and $\lambda_2$ and minimize

\begin{displaymath}\lambda_1\alpha + \lambda_2 E_{\mu_0}[(\bar{X} - \mu_0)\phi(X)] + \beta
\end{displaymath}

The quantity in question is just

\begin{displaymath}\int [\phi(x) f_0(x)(\lambda_1+\lambda_2(\bar{X} - \mu_0)) +
(1-\phi(x))f_1(x)] dx
\end{displaymath}

As before this is minimized by

\begin{displaymath}\phi(x) =\left\{\begin{array}{ll}
1 & \frac{f_1(x)}{f_0(x)} >...
...(x)} < \lambda_1+\lambda_2(\bar{X} - \mu_0)
\end{array}\right.
\end{displaymath}

The likelihood ratio f1/f0 is simply

\begin{displaymath}\exp\{ n(\mu_1-\mu_0)\bar{X} + n(\mu_0^2-\mu_1^2)/2\}
\end{displaymath}

and this exceeds the linear function

\begin{displaymath}\lambda_1+\lambda_2(\bar{X} - \mu_0)
\end{displaymath}

for all $\bar{X}$ sufficiently large or small. That is, the quantity

\begin{displaymath}\lambda_1\alpha + \lambda_2 E_{\mu_0}[(\bar{X} - \mu_0)\phi(X)] + \beta
\end{displaymath}

is minimized by a rejection region of the form

\begin{displaymath}\{\bar{X} > K_U\} \cup \{ \bar{X} < K_L\}
\end{displaymath}

To satisfy the constraints we adjust KU and KL to get level $\alpha$ and $\pi^\prime(\mu_0) = 0$. The second condition shows that the rejection region is symmetric about $\mu_0$ and then we discover that the test rejects for

\begin{displaymath}\sqrt{n}\vert\bar{X} - \mu_0\vert > z_{\alpha/2}
\end{displaymath}

Now you have to mimic the Neyman Pearson lemma proof to check that if $\lambda_1$ and $\lambda_2$ are adjusted so that the unconstrained problem has the rejection region given then the resulting test minimizes $\beta$ subject to the two constraints.

A test $\phi^*$ is a Uniformly Most Powerful Unbiased level $\alpha_0$test if

1.
$\phi^*$ has level $\alpha \le \alpha_0$.

2.
$\phi^*$ is unbiased.

3.
If $\phi$ has level $\alpha \le \alpha_0$ then for every $\theta\in \Theta_1$ we have

\begin{displaymath}E_\theta(\phi(X)) \le E_\theta(\phi^*(X))
\end{displaymath}

Conclusion: The two sided z test which rejects if

\begin{displaymath}\vert Z\vert > z_{\alpha/2}
\end{displaymath}

where

\begin{displaymath}Z=n^{1/2}(\bar{X} -\mu_0)
\end{displaymath}

is the uniformly most powerful unbiased test of $\mu=\mu_0$against the two sided alternative $\mu\neq\mu_0$.

Nuisance Parameters

What good can be said about the t-test? It's UMPU.

Suppose $X_1,\ldots,X_n$ are iid $N(\mu,\sigma^2)$ and that we want to test $\mu=\mu_0$ or $\mu \le \mu_0$ against $\mu>\mu_0$. Notice that the parameter space is two dimensional and that the boundary between the null and alternatives is

\begin{displaymath}\{(\mu,\sigma); \mu=\mu_0,\sigma>0\}
\end{displaymath}

If a test has $\pi(\mu,\sigma) \le \alpha$ for all $\mu \le \mu_0$ and $\pi(\mu,\sigma) \ge \alpha$ for all $\mu>\mu_0$ then we must have $\pi(\mu_0,\sigma) =\alpha$for all $\sigma$ because the power function of any test must be conntinuous. (This actually uses the dominated convergence theorem; the power function is an integral.)

Now think of $\{(\mu,\sigma);\mu=\mu_0\}$ as a parameter space for a model. For this parameter space you can check that

\begin{displaymath}S=\sum(X_i-\mu_0)^2
\end{displaymath}

is a complete sufficient statistic. Remember that the definitions of both completeness and sufficiency depend on the parameter space. Suppose $\phi(\sum X_i, S)$is an unbiased level $\alpha$ test. Then we have

\begin{displaymath}E_{\mu_0,\sigma}(\phi(\sum X_i, S)) = \alpha
\end{displaymath}

for all $\sigma$. Condition on S and get

\begin{displaymath}E_{\mu_0,\sigma}[E(\phi(\sum X_i, S)\vert S)] = \alpha
\end{displaymath}

for all $\sigma$. Sufficiency guarantees that

\begin{displaymath}g(S) = E(\phi(\sum X_i, S)\vert S)
\end{displaymath}

is a statistic and completeness that

\begin{displaymath}g(S) \equiv \alpha
\end{displaymath}

Now let us fix a single value of $\sigma$ and a $\mu_1>\mu_0$. To make our notation simpler I take $\mu_0=0$. Our observations above permit us to condition on S=s. Given S=s we have a level $\alpha$ test which is a function of $\bar{X}$.

If we maximize the conditional power of this test for each s then we will maximize its power. What is the conditional model given S=s? That is, what is the conditional distribution of $\bar{X}$ given S=s? The answer is that the joint density of $\bar{X},S$ is of the form

\begin{displaymath}f_{\bar{X},S}(t,s) = h(s,t) \exp\{\theta_1 t + \theta_2 s +c(\theta_1,\theta_2)\}
\end{displaymath}

where $\theta_1=n\mu/\sigma^2$ and $\theta_2 = -1/\sigma^2$.

This makes the conditional density of $\bar{X}$ given S=s of the form

\begin{displaymath}f_{\bar{X}\vert s}(t\vert s) = h(s,t)\exp\{\theta_1 t+c^*(\theta_1,s)\}
\end{displaymath}

Notice the disappearance of $\theta_2$. Notice that the null hypothesis is actually $\theta_1=0$. This permits the application of the NP lemma to the conditional family to prove that the UMP unbiased test must have the form

\begin{displaymath}\phi(\bar{X},S) = 1(\bar{X} > K(S))
\end{displaymath}

where K(S) is chosen to make the conditional level $\alpha$. The function $x\mapsto x/\sqrt{a-x^2}$ is increasing in x for each a so that we can rewrite $\phi$ in the form

\begin{displaymath}\phi(\bar{X},S) = 1(n^{1/2}\bar{X}/\sqrt{n[S/n-\bar{X}^2]/(n-1)} > K^*(S))
\end{displaymath}

for some K*. The quantity

\begin{displaymath}T=\frac{n^{1/2}\bar{X}}{\sqrt{n[S/n-\bar{X}^2]/(n-1)}}
\end{displaymath}

is the usual t statistic and is exactly independent of S (see Theorem 6.1.5 on page 262 in Casella and Berger). This guarantees that

\begin{displaymath}K^*(S) = t_{n-1,\alpha}
\end{displaymath}

and makes our UMPU test the usual t test.


next up previous



Richard Lockhart
2000-03-16